Completed rec9 solutions
[course.git] / latex / problems / Young_and_Freedman_12 / problem28.30.tex
1 % Commented out because these are defined in problem28.12.tex
2 %\newcommand{\dB}{d\vect{B}}
3 %\newcommand{\dl}{d\vect{l}}
4
5 \begin{problem*}{28.30}
6 Calculate the magnitude and direction of the magnetic field at point
7 $P$ due to the current in the semicircular section of wire shown in
8 Fig.~28.46.  (\emph{Hint:} Does the current in the long, straight
9 section of the wire produce any field at $P$?)
10 \begin{center}
11 \begin{asy}
12 import Mechanics;
13 import Circ; // TODO: wire(path);
14
15 real u = 1cm;
16 real r = u;
17
18 Distance R = Distance((0,0), (r*Cos(45),r*Sin(45)), "$r$");
19 R.draw();
20 draw((-2r,0)--(-r,0){N}..(0,r){E}..{S}(r,0)--(2r,0));
21 TwoTerminal Il = current((-1.5r,0), "I");
22 TwoTerminal Ir = current((1.5r,0), "I");
23 dot("$P$", (0,0), S);
24 \end{asy}
25 \end{center}
26 \end{problem*}
27
28 \begin{solution}
29 The Biot-Savart law
30 \begin{equation}
31   \dB = \frac{\mu_0}{4\pi}\cdot\frac{I\dl\times\rhat}{r^2}
32 \end{equation}
33 gives the magnetic field from an infinitesimal chunk of current.
34 The straight sections of wire do not contribute any magnetic field at $P$, because
35 \begin{equation}
36   |\dl\times\rhat| = |\dl|\cdot|\rhat|\sin\theta = 0 \;,
37 \end{equation}
38 since $\theta=0\dg$ to the left and $180\dg$ to the right.
39
40 For all portions of wire along the semicircle, $\theta=90\dg$, and
41 $\dl\times\rhat$ points into the page.  The net magnetic field at $P$
42 is therefore
43 \begin{equation}
44   B = \int_s \frac{\mu_0}{4\pi}\cdot\frac{I\dl\times\rhat}{r^2}  
45     = \frac{\mu_0}{4\pi}\cdot\frac{I}{r^2} \int_s |\dl|\cdot|\rhat|\sin(90\dg)  
46     = \frac{\mu_0 I}{4\pi r^2} \int_s |\dl|
47     = \frac{\mu_0 I}{4\pi r^2} \cdot \pi r
48     = \ans{\frac{\mu_0I}{4r}} \;.
49 \end{equation}
50 where $\int_s |\dl|$ is just the length of the semicircle, which is
51 half the $2\pi r$ circumference of the entire circle.
52 \end{solution}